Q. Solving a Quadratic Recurrence Relation: Finding Coefficients a, b, and c

Click For Summary
To solve the quadratic recurrence relation given by un = a + bn + cn² with initial conditions u1 = 0, u2 = 3, and u3 = 12, a system of three equations is established: a + b + c = 0, a + 2b + 4c = 3, and a + 3b + 9c = 12. The coefficients a, b, and c can be determined by solving this system, which does not require the quadratic formula. The solution yields a = 3, b = -6, and c = 3. This approach emphasizes the importance of setting up and solving simultaneous equations in recurrence relations.
odolwa99
Messages
85
Reaction score
0

Homework Statement



Dang it! More recurrence relation problems, but this time it’s due to a quadratic equation.

Q. In the sequence u1, u2, u3,…,un,
u1 = 0, u2 = 3, u3 = 12 and un = a + bn + cn2
Find the values of a, b and c.

Homework Equations



Provided at back of book…
Answer: a = 3, b = -6, c = 3

The Attempt at a Solution



Attempt:
If n = 1 then u1 = a + b(1) + c(1)2 = 0
= a + b + c = 0
If n = 2 then u2 = a + b(2) + c(2)2 = 3
= a + 2b + 4c = 3
If n = 3 then u3 = a + b(3) + c(3)2 = 12
= a + 3b + 9c = 12

Each value of un is a multiple of 3, hence the answers (a, b and c) are also multiples of 3, so I’m guessing that I need to use some kind of ratio solution between each new quadratic to find the answer. The difficulty I’m having is that other quadratic equations I’ve solved for already had values for the coefficients, so I’m uncertain on what quadratic formula I need at this point. I probably don’t need one, but I’m definitely stuck either way.
 
Physics news on Phys.org
Hi odolwa99! :smile:

There is no need for quadratic formula's. You have a system of three equations and three unknowns:

a + b + c = 0
a + 2b + 4c = 3
a + 3b + 9c = 12

Solve it.
 
Ok, thanks for the tip. I'll give it a second look.
 
Question: A clock's minute hand has length 4 and its hour hand has length 3. What is the distance between the tips at the moment when it is increasing most rapidly?(Putnam Exam Question) Answer: Making assumption that both the hands moves at constant angular velocities, the answer is ## \sqrt{7} .## But don't you think this assumption is somewhat doubtful and wrong?

Similar threads

  • · Replies 5 ·
Replies
5
Views
2K
  • · Replies 5 ·
Replies
5
Views
1K
  • · Replies 13 ·
Replies
13
Views
2K
Replies
31
Views
4K
  • · Replies 6 ·
Replies
6
Views
3K
Replies
8
Views
3K
Replies
8
Views
3K
  • · Replies 1 ·
Replies
1
Views
2K
  • · Replies 2 ·
Replies
2
Views
2K
  • · Replies 8 ·
Replies
8
Views
3K